LSAT and Law School Admissions Forum

Get expert LSAT preparation and law school admissions advice from PowerScore Test Preparation.

 Administrator
PowerScore Staff
  • PowerScore Staff
  • Posts: 8916
  • Joined: Feb 02, 2011
|
#27450
Complete Question Explanation

Justify the Conclusion—SN. The correct answer choice is (C)

The conclusion here is that some of Bell’s subsidiaries will be sold. This is said to follow from the idea that if there is no strike then management must have increased wages, as increased wages would require management to sell some of its subsidiaries:
  • No Strike :arrow: Inc Wages :arrow: Sell Subs
To prove the end of the chain (sell subs), simply state that the beginning of the chain will occur (no strike).

Answer choice (A): This does not initiate the chain relationship shown above.

Answer choice (B): We want management to increase wages, as that would prove the conclusion.

Answer choice (C): This is the correct answer choice. As mentioned above, this would initiate the chain relationship described in the stimulus.

Answer choice (D): Just because the president has the authority to increase wages does not mean that it will happen.

Answer choice (E): Again, this does not show that the wage increase will occur, so it does not prove the conclusion.
 wapet1
  • Posts: 9
  • Joined: Sep 13, 2018
|
#60910
hi i have quick question! if there was an answer choice saying “wage would increase” would this be a correct answer as well?

thank you
 Brook Miscoski
PowerScore Staff
  • PowerScore Staff
  • Posts: 418
  • Joined: Sep 13, 2018
|
#60921
Wapet,

The second sentence establishes the following relationship:

Wages Increase :arrow: Sell Subsidiaries.

Therefore, if we were told that wages would increase, yes, we would know that Bell must sell some of its subsidiaries.

It would be odd for the choice not to incorporate the full conditional reasoning of the stimulus, but you are right that the conclusion would follow.

The LSAT test writers will not give you two correct choices, so if they gave you that choice, it would replace (C), the correct choice.
 wapet1
  • Posts: 9
  • Joined: Sep 13, 2018
|
#60944
Brook Miscoski wrote:Wapet,

The second sentence establishes the following relationship:

Wages Increase :arrow: Sell Subsidiaries.

Therefore, if we were told that wages would increase, yes, we would know that Bell must sell some of its subsidiaries.

It would be odd for the choice not to incorporate the full conditional reasoning of the stimulus, but you are right that the conclusion would follow.

The LSAT test writers will not give you two correct choices, so if they gave you that choice, it would replace (C), the correct choice.

thank you so much for the reply. :)
 oli_oops
  • Posts: 37
  • Joined: Aug 22, 2018
|
#62303
Administrator wrote:Complete Question Explanation

Justify the Conclusion—SN. The correct answer choice is (C)

The conclusion here is that some of Bell’s subsidiaries will be sold. This is said to follow from the idea that if there is no strike then management must have increased wages, as increased wages would require management to sell some of its subsidiaries:
  • No Strike :arrow: Inc Wages :arrow: Sell Subs
To prove the end of the chain (sell subs), simply state that the beginning of the chain will occur (no strike).

Answer choice (A): This does not initiate the chain relationship shown above.

Answer choice (B): We want management to increase wages, as that would prove the conclusion.

Answer choice (C): This is the correct answer choice. As mentioned above, this would initiate the chain relationship described in the stimulus.

Answer choice (D): Just because the president has the authority to increase wages does not mean that it will happen.

Answer choice (E): Again, this does not show that the wage increase will occur, so it does not prove the conclusion.

Hello!

Thank you for your simple and straightforward explanation, it helped a lot.
I'm just curious to know that is this a common form of Justify the Conclusion question? Or is it just me?
It is as simple as you explained it/as it is, however, I was very thrown off by the form.
 Jay Donnell
PowerScore Staff
  • PowerScore Staff
  • Posts: 144
  • Joined: Jan 09, 2019
|
#62320
Hi Oli!

As far as Justify questions go, this one is just a little bit peculiar in that the missing assumption provided by the correct response was in the form of an absolute statement rather than a conditional one.

Very often, conditional-heavy Justify questions take one of the following standard forms:

1)
A --> B
( )
_______
A --> C

Answer provides missing link of: B --> C

2)

( )
B --> C
______
A --> C

Answer provides the missing link of: A --> B

3)

A --> B
( )
C --> D
______
A --> D

Answer provides the missing link of: B --> C

3)

A
( )
_______
B

Answer provides the missing link of: A --> B



This question however, ended up (after taking some contrapositives in the premises) looking like:

A --> B
B --> C
_______
C

This means that we need an absolute fact to activate the conditional premises, so really the correct response could have worked if it provided (A) or (B), as each fact could result in the guaranteeing of (C) in the conclusion.

In terms of this question's topic, the argument looks like this:

~S --> IW
IW --> SS
________
SS

So the answer could have provided (~S) aka "no strike" or (IW) aka "increased wages", as either would have guaranteed that Bell would be forced to (SS) aka sell some subsidiaries.


Not every Justify question matches one of the above structures, but the vast majority will, so I hope this helps give you some things to stay on the lookout for!
 LSAT2020
  • Posts: 31
  • Joined: Jun 24, 2020
|
#76502
Would we say that this follows the logic of A->B-->C with the valid conclusion being A-->C? If so, does this mean that the conclusion from the stimulus is incomplete, in the sense that it's missing the A going into the C. It seems that we were only given the C part of the conclusion. This is what I mean:

A= No strike

B= Increase Wage

C= Sell Sub


Premise: No strike--->Increase Wage--->Sell Sub

Conclusion: Sell Sub
 Adam Tyson
PowerScore Staff
  • PowerScore Staff
  • Posts: 5153
  • Joined: Apr 14, 2011
|
#76556
Exactly right, LSAT2020! To prove that conclusion of C (sell subsidiaries) we need to know that a sufficient condition is going to happen, such as A (no strike). Well done!
 Coleman
  • Posts: 44
  • Joined: Jul 07, 2020
|
#76841
I was hesitating over between C and D and here's my logic for why C is not sufficient to tell "some of Bell's subsidiaries will be sold."

In the passage, in order to increase the wages, they have to sell off some subsidiaries. However, selling subsidiaries is not sufficient condition that guarantees a wage increase will take place. What if selling some subsidiaries are merely necessary conditions to increase wages, BUT still not sufficient? What if we need an additional step in addition to selling subsidiaries in order to increase wages?
 Rachael Wilkenfeld
PowerScore Staff
  • PowerScore Staff
  • Posts: 1358
  • Joined: Dec 15, 2011
|
#76895
Hi Coleman,

It seems like you might be flipping around sufficient and necessary a bit. You are right that selling subsidiaries is not a sufficient condition here. But we don't need it to be. It's a necessary condition, and this justify question is asking us to find what is sufficient for that necessary condition.

To figure that out we have a standard conditional chain. If workers don't strike, then the company must increase wages. If the company increases wages, then some subsidiaries must be sold. We can draw that out like this

Workers strike :arrow: increase wages :arrow: sell subsidiaries.

Our conclusion is the end of that chain: The company will sell the subsidiaries.

What would require the subsidiaries to be sold? Either of the conditions further back on the chain. If the workers don't strike, then the company would have increased wages, at which point we can get to the conclusion that the subsidies will be sold. If the company increases wages, then it had to sell subsidiaries. Either the workers not striking or the increase in wages get us to the conclusion in the stimulus. That's why we pick answer choice (C) here.

Answer choice (D) however is not enough to get to our conclusion. Even if the president has the authority to authorize an increase in wages, it doesn't tell us if he DOES increase the wages. We need the wage increase to occur, not just the authority to grant the increase.

Hope that helps!
Rachael

Get the most out of your LSAT Prep Plus subscription.

Analyze and track your performance with our Testing and Analytics Package.